Use Fourier analysis to construct an amplitude spectrum

Click For Summary
The discussion revolves around constructing an amplitude spectrum for the signal y(t) = |sin(120πt)| using Fourier analysis. The period of the function is determined to be T = 1/60, derived from the formula for the period of sine functions. Participants clarify that the absolute value does not affect the use of half-range formulas, which can simplify the integration process. There is confusion regarding the integration limits and whether the function is odd, but it is confirmed that the function remains even when using half-range cosine formulas. The conversation emphasizes the importance of understanding the properties of the function for accurate Fourier analysis.
luckyduck
Messages
7
Reaction score
0

Homework Statement



A sensor yields a signal y(t) = |sin(120\pit)|
a. Using Fourier analysis please construct an amplitude spectrum for this signal.

Homework Equations


A0 = \frac{1}{T}\int ^{-T/2}_{T/2}y(t) dt
An =\frac{2}{T}\int^{-T/2}_{T/2}y(t)cos\frac{2n\pi t}{T}dt

The Attempt at a Solution


Because y(-t) = y(t), the function is even, and we can ignore Bn.

My question is: what is the period? In prior examples, we used T = 2\pi.
Therefore, will:
A0 = \frac{1}{2\pi} \int^{-\pi}_{\pi} |sin(120\pit)| dt?
 
Physics news on Phys.org
luckyduck said:

Homework Statement



A sensor yields a signal y(t) = |sin(120\pit)|
a. Using Fourier analysis please construct an amplitude spectrum for this signal.

Homework Equations


A0 = \frac{1}{T}\int ^{-T/2}_{T/2}y(t) dt
An =\frac{2}{T}\int^{-T/2}_{T/2}y(t)cos\frac{2n\pi t}{T}dt

The Attempt at a Solution


Because y(-t) = y(t), the function is even, and we can ignore Bn.

My question is: what is the period? In prior examples, we used T = 2\pi.
Therefore, will:
A0 = \frac{1}{2\pi} \int^{-\pi}_{\pi} |sin(120\pit)| dt?

The period of ##\sin(bt)## is ##2\pi /b## so the period of your function without the absolute values would be$$
T=\frac{2\pi}{120\pi}=\frac 1 {60}$$I would use that in half range formula. Note that by using the half range formula you can drop the absolute value.
 
Last edited:
Thank you!

By half range, do you mean integrating between 0 and 30?
 
Also, would this mean that the function is now odd?
 
luckyduck said:
Thank you!

By half range, do you mean integrating between 0 and 30?

No. Half the period would be 1/120.

luckyduck said:
Also, would this mean that the function is now odd?

No. You use the half range cosine formulas which gives the even extension of the function, which has the same effect as the absolute values. Look at the half range formulas in your text.
 
Duh! Brain fart. Sorry, I don't have a text for this class, so I've been relying on google. Thanks again for your help!
 
Question: A clock's minute hand has length 4 and its hour hand has length 3. What is the distance between the tips at the moment when it is increasing most rapidly?(Putnam Exam Question) Answer: Making assumption that both the hands moves at constant angular velocities, the answer is ## \sqrt{7} .## But don't you think this assumption is somewhat doubtful and wrong?

Similar threads

  • · Replies 16 ·
Replies
16
Views
2K
  • · Replies 1 ·
Replies
1
Views
2K
  • · Replies 2 ·
Replies
2
Views
1K
Replies
1
Views
2K
  • · Replies 1 ·
Replies
1
Views
1K
  • · Replies 3 ·
Replies
3
Views
2K
  • · Replies 3 ·
Replies
3
Views
2K
  • · Replies 8 ·
Replies
8
Views
3K
  • · Replies 3 ·
Replies
3
Views
2K
  • · Replies 11 ·
Replies
11
Views
3K